Rudin exercise 3.8 solutionShowing a series converges absolutely iff it has a uniform boundIf $sum a_n$ is...
Purchasing a ticket for someone else in another country?
Arithmetic mean geometric mean inequality unclear
How to pronounce the slash sign
Large drywall patch supports
Tiptoe or tiphoof? Adjusting words to better fit fantasy races
Valid Badminton Score?
What is the best translation for "slot" in the context of multiplayer video games?
Why escape if the_content isnt?
Would a high gravity rocky planet be guaranteed to have an atmosphere?
How does it work when somebody invests in my business?
Is this apparent Class Action settlement a spam message?
Is there a korbon needed for conversion?
Would this custom Sorcerer variant that can only learn any verbal-component-only spell be unbalanced?
Is there a good way to store credentials outside of a password manager?
How to Reset Passwords on Multiple Websites Easily?
How do I go from 300 unfinished/half written blog posts, to published posts?
Proof of work - lottery approach
Escape a backup date in a file name
Different result between scanning in Epson's "color negative film" mode and scanning in positive -> invert curve in post?
Did Dumbledore lie to Harry about how long he had James Potter's invisibility cloak when he was examining it? If so, why?
Roman Numeral Treatment of Suspensions
How did Arya survive the stabbing?
How can a function with a hole (removable discontinuity) equal a function with no hole?
when is out of tune ok?
Rudin exercise 3.8 solution
Showing a series converges absolutely iff it has a uniform boundIf $sum a_n$ is convergent, with $a_n geq 0$ $forall n in mathbb{N}$, show that if $p > 1$, $sum a_n^p$ converges.Why doesn't this work for Rudin Exercise 3.8Convergence of recursive sequence $a_{n+1} =frac{ 1}{k} left(a_{n} + frac{k}{a_{n}}right)$Two p-norms are not equivalent for different p on $ell_1$Sum of squares converges given sum doesWhat's significant in Abel's Theorem Proof in Baby Rudin?Why is my proof wrong? (Rudin Ch 3 #8) If $sum a_n$ converges, and if ${b_n}$ is monotonic and bounded, prove that $sum a_n b_n$ converges.Calculus-probabilityProof Verification: A monotonically increasing sequence that is bounded above always has a LUB
$begingroup$
Problem . if the sequence $sumlimits_{}{a_{n}}$ converges and $b_{n}$ is a monotonic bounded sequence then $sumlimits_{} {a_{n}b_{n}}$ converges.
Solution. Let's use the identity:
$sumlimits_{n=p}^q {a_{n}b_{n}}$ = $sumlimits_{n=p}^{q-1} {s_{n}(b_{n}-b_{n-1})}+s_{q}b_{q}-s_{p-1}b_{p}$, where $s_{n}=sumlimits_{k=1}^n {a_{n}}$. Then $|sumlimits_{n=p}^{q-1} {s_{n}(b_{n}-b_{n-1})}|<=|sumlimits_{n=p}^{q-1} {s_{n}}|B$, where B is the difference of upper and lower bound of the sequence $b_{n}$. since $a_{n}$ is convergent we can make $|sumlimits_{n=p}^{q-1} {s_{n}}|$ as small as we want and therefore we can make this $|sumlimits_{n=p}^{q-1} {s_{n}(b_{n}-b_{n-1})}|$ as small as we want like $x/3$. Then let's take the absolute value of another term of the right side of the identity.
$|s_{q}b_{q}|<=|s_{q}||b_{up}|$, where $b_{up}$ is the upper bound of the sequence and the same argument here too. we can make this term as small as we want. The same goes to the absolute value of the third term on the right side of the identity. Finally $|sumlimits_{n=p}^{q} {a_{n}b_{n}}|<=|sumlimits_{n=p}^{q-1} {s_{n}(b_{n}-b_{n-1})}|+|s_{q}b_{q}|+|s_{p-1}b_{p}|$. We can find $p$ and $q$ to make the right side small enough, completing the proof. Is this right? I am not satisfied what I have written here, I should have stated better in my opinion but the general idea of the proof is shown I suppose.
real-analysis proof-verification
$endgroup$
add a comment |
$begingroup$
Problem . if the sequence $sumlimits_{}{a_{n}}$ converges and $b_{n}$ is a monotonic bounded sequence then $sumlimits_{} {a_{n}b_{n}}$ converges.
Solution. Let's use the identity:
$sumlimits_{n=p}^q {a_{n}b_{n}}$ = $sumlimits_{n=p}^{q-1} {s_{n}(b_{n}-b_{n-1})}+s_{q}b_{q}-s_{p-1}b_{p}$, where $s_{n}=sumlimits_{k=1}^n {a_{n}}$. Then $|sumlimits_{n=p}^{q-1} {s_{n}(b_{n}-b_{n-1})}|<=|sumlimits_{n=p}^{q-1} {s_{n}}|B$, where B is the difference of upper and lower bound of the sequence $b_{n}$. since $a_{n}$ is convergent we can make $|sumlimits_{n=p}^{q-1} {s_{n}}|$ as small as we want and therefore we can make this $|sumlimits_{n=p}^{q-1} {s_{n}(b_{n}-b_{n-1})}|$ as small as we want like $x/3$. Then let's take the absolute value of another term of the right side of the identity.
$|s_{q}b_{q}|<=|s_{q}||b_{up}|$, where $b_{up}$ is the upper bound of the sequence and the same argument here too. we can make this term as small as we want. The same goes to the absolute value of the third term on the right side of the identity. Finally $|sumlimits_{n=p}^{q} {a_{n}b_{n}}|<=|sumlimits_{n=p}^{q-1} {s_{n}(b_{n}-b_{n-1})}|+|s_{q}b_{q}|+|s_{p-1}b_{p}|$. We can find $p$ and $q$ to make the right side small enough, completing the proof. Is this right? I am not satisfied what I have written here, I should have stated better in my opinion but the general idea of the proof is shown I suppose.
real-analysis proof-verification
$endgroup$
$begingroup$
yeah, "making as small as we want" is a bad idea, but I can say that I can bound that sum by taking appropriate $p$ and $q$.
$endgroup$
– shota kobakhidze
Mar 15 at 18:28
add a comment |
$begingroup$
Problem . if the sequence $sumlimits_{}{a_{n}}$ converges and $b_{n}$ is a monotonic bounded sequence then $sumlimits_{} {a_{n}b_{n}}$ converges.
Solution. Let's use the identity:
$sumlimits_{n=p}^q {a_{n}b_{n}}$ = $sumlimits_{n=p}^{q-1} {s_{n}(b_{n}-b_{n-1})}+s_{q}b_{q}-s_{p-1}b_{p}$, where $s_{n}=sumlimits_{k=1}^n {a_{n}}$. Then $|sumlimits_{n=p}^{q-1} {s_{n}(b_{n}-b_{n-1})}|<=|sumlimits_{n=p}^{q-1} {s_{n}}|B$, where B is the difference of upper and lower bound of the sequence $b_{n}$. since $a_{n}$ is convergent we can make $|sumlimits_{n=p}^{q-1} {s_{n}}|$ as small as we want and therefore we can make this $|sumlimits_{n=p}^{q-1} {s_{n}(b_{n}-b_{n-1})}|$ as small as we want like $x/3$. Then let's take the absolute value of another term of the right side of the identity.
$|s_{q}b_{q}|<=|s_{q}||b_{up}|$, where $b_{up}$ is the upper bound of the sequence and the same argument here too. we can make this term as small as we want. The same goes to the absolute value of the third term on the right side of the identity. Finally $|sumlimits_{n=p}^{q} {a_{n}b_{n}}|<=|sumlimits_{n=p}^{q-1} {s_{n}(b_{n}-b_{n-1})}|+|s_{q}b_{q}|+|s_{p-1}b_{p}|$. We can find $p$ and $q$ to make the right side small enough, completing the proof. Is this right? I am not satisfied what I have written here, I should have stated better in my opinion but the general idea of the proof is shown I suppose.
real-analysis proof-verification
$endgroup$
Problem . if the sequence $sumlimits_{}{a_{n}}$ converges and $b_{n}$ is a monotonic bounded sequence then $sumlimits_{} {a_{n}b_{n}}$ converges.
Solution. Let's use the identity:
$sumlimits_{n=p}^q {a_{n}b_{n}}$ = $sumlimits_{n=p}^{q-1} {s_{n}(b_{n}-b_{n-1})}+s_{q}b_{q}-s_{p-1}b_{p}$, where $s_{n}=sumlimits_{k=1}^n {a_{n}}$. Then $|sumlimits_{n=p}^{q-1} {s_{n}(b_{n}-b_{n-1})}|<=|sumlimits_{n=p}^{q-1} {s_{n}}|B$, where B is the difference of upper and lower bound of the sequence $b_{n}$. since $a_{n}$ is convergent we can make $|sumlimits_{n=p}^{q-1} {s_{n}}|$ as small as we want and therefore we can make this $|sumlimits_{n=p}^{q-1} {s_{n}(b_{n}-b_{n-1})}|$ as small as we want like $x/3$. Then let's take the absolute value of another term of the right side of the identity.
$|s_{q}b_{q}|<=|s_{q}||b_{up}|$, where $b_{up}$ is the upper bound of the sequence and the same argument here too. we can make this term as small as we want. The same goes to the absolute value of the third term on the right side of the identity. Finally $|sumlimits_{n=p}^{q} {a_{n}b_{n}}|<=|sumlimits_{n=p}^{q-1} {s_{n}(b_{n}-b_{n-1})}|+|s_{q}b_{q}|+|s_{p-1}b_{p}|$. We can find $p$ and $q$ to make the right side small enough, completing the proof. Is this right? I am not satisfied what I have written here, I should have stated better in my opinion but the general idea of the proof is shown I suppose.
real-analysis proof-verification
real-analysis proof-verification
edited Mar 15 at 18:05
Alex Provost
15.6k22351
15.6k22351
asked Mar 15 at 17:59
shota kobakhidzeshota kobakhidze
115
115
$begingroup$
yeah, "making as small as we want" is a bad idea, but I can say that I can bound that sum by taking appropriate $p$ and $q$.
$endgroup$
– shota kobakhidze
Mar 15 at 18:28
add a comment |
$begingroup$
yeah, "making as small as we want" is a bad idea, but I can say that I can bound that sum by taking appropriate $p$ and $q$.
$endgroup$
– shota kobakhidze
Mar 15 at 18:28
$begingroup$
yeah, "making as small as we want" is a bad idea, but I can say that I can bound that sum by taking appropriate $p$ and $q$.
$endgroup$
– shota kobakhidze
Mar 15 at 18:28
$begingroup$
yeah, "making as small as we want" is a bad idea, but I can say that I can bound that sum by taking appropriate $p$ and $q$.
$endgroup$
– shota kobakhidze
Mar 15 at 18:28
add a comment |
2 Answers
2
active
oldest
votes
$begingroup$
Assume that ${b_n}$ is increasing. If $s_n=sum_{k=1}^n a_k$, then
$$
sum_{k=1}^n a_kb_k=sum_{k=1}^n (s_k-s_{k-1})b_k=sum_{k=1}^n s_kb_k-sum_{k=2}^n s_{k-1}b_k=s_nb_n+
sum_{k=2}^{n}s_{k-1}(b_{k-1}-b_{k}).
$$
Now ${s_nb_n}$ is convergent, as a product of convergent sequences, while the series
$sum s_{n-1}(b_{n-1}-b_{n})$
is dominated by the series $sum M(b_{n}-b_{n-1})$, where $M=sup_{ninmathbb N}|s_n|$, and since ${b_{n}-b_{n-1}}$ is absolutely summable,
so is $sum s_{n-1}(b_{n-1}-b_{n})$.
$endgroup$
$begingroup$
yeah, it looks like a similar idea, stated brilliantly.
$endgroup$
– shota kobakhidze
Mar 15 at 19:11
add a comment |
$begingroup$
The general idea is correct. The expression “the upper bound of the sequence” is not appropriate (there are infinitely many upper bounds). I think that it would be better to say that $b_{up}$ is an upper bound of the sequence $bigl(lvert b_nrvertbigr)_{ninmathbb N}$. With this choice, $b_{up}$ is non-negative. Also, with this choice you have$$leftlvertsum_{n=p}^{q-1}s_n(b_n-b_{n-1})rightrvertleqslant2leftlvertsum_{n=p}^{q-1}s_nrightrvert b_{up}.$$Again, the expression “the difference of upper and lower bound of the sequence $b_n$” is not appropriate.
$endgroup$
$begingroup$
yeah, understood. Thanks for the response. By the way, saying that "we can make it as small as we want" is incorrect right? It would be more correct to say "bounded". Then the whole sum becomes "bounded" and not "as small as I wanted".
$endgroup$
– shota kobakhidze
Mar 15 at 18:38
$begingroup$
Oops! You are right. What you shoud use at that point is that $(s_n)_{ninmathbb N}$ is a boounded sequence and that the series $sum_{n=1}^inftylvert b_n-b_{n+1}rvert$ converges (this follows from the fact that $(b_n)_{ninmathbb N}$ is a monotonic and bounded sequence).
$endgroup$
– José Carlos Santos
Mar 15 at 18:47
$begingroup$
By the way, is it necessary for $b_{n}$ to be monotonic? cause I haven't used that fact, I just used the boundedness.
$endgroup$
– shota kobakhidze
Mar 15 at 23:16
$begingroup$
See my previous comment. Without that sequence being monotonic, how do you know that the series $sum_{n=1}^inftylvert b_n-b_{n+1}rvert$ converges?
$endgroup$
– José Carlos Santos
Mar 15 at 23:33
$begingroup$
yes, I see. It was my bad. Thanks :)
$endgroup$
– shota kobakhidze
Mar 16 at 0:00
add a comment |
Your Answer
StackExchange.ifUsing("editor", function () {
return StackExchange.using("mathjaxEditing", function () {
StackExchange.MarkdownEditor.creationCallbacks.add(function (editor, postfix) {
StackExchange.mathjaxEditing.prepareWmdForMathJax(editor, postfix, [["$", "$"], ["\\(","\\)"]]);
});
});
}, "mathjax-editing");
StackExchange.ready(function() {
var channelOptions = {
tags: "".split(" "),
id: "69"
};
initTagRenderer("".split(" "), "".split(" "), channelOptions);
StackExchange.using("externalEditor", function() {
// Have to fire editor after snippets, if snippets enabled
if (StackExchange.settings.snippets.snippetsEnabled) {
StackExchange.using("snippets", function() {
createEditor();
});
}
else {
createEditor();
}
});
function createEditor() {
StackExchange.prepareEditor({
heartbeatType: 'answer',
autoActivateHeartbeat: false,
convertImagesToLinks: true,
noModals: true,
showLowRepImageUploadWarning: true,
reputationToPostImages: 10,
bindNavPrevention: true,
postfix: "",
imageUploader: {
brandingHtml: "Powered by u003ca class="icon-imgur-white" href="https://imgur.com/"u003eu003c/au003e",
contentPolicyHtml: "User contributions licensed under u003ca href="https://creativecommons.org/licenses/by-sa/3.0/"u003ecc by-sa 3.0 with attribution requiredu003c/au003e u003ca href="https://stackoverflow.com/legal/content-policy"u003e(content policy)u003c/au003e",
allowUrls: true
},
noCode: true, onDemand: true,
discardSelector: ".discard-answer"
,immediatelyShowMarkdownHelp:true
});
}
});
Sign up or log in
StackExchange.ready(function () {
StackExchange.helpers.onClickDraftSave('#login-link');
});
Sign up using Google
Sign up using Facebook
Sign up using Email and Password
Post as a guest
Required, but never shown
StackExchange.ready(
function () {
StackExchange.openid.initPostLogin('.new-post-login', 'https%3a%2f%2fmath.stackexchange.com%2fquestions%2f3149615%2frudin-exercise-3-8-solution%23new-answer', 'question_page');
}
);
Post as a guest
Required, but never shown
2 Answers
2
active
oldest
votes
2 Answers
2
active
oldest
votes
active
oldest
votes
active
oldest
votes
$begingroup$
Assume that ${b_n}$ is increasing. If $s_n=sum_{k=1}^n a_k$, then
$$
sum_{k=1}^n a_kb_k=sum_{k=1}^n (s_k-s_{k-1})b_k=sum_{k=1}^n s_kb_k-sum_{k=2}^n s_{k-1}b_k=s_nb_n+
sum_{k=2}^{n}s_{k-1}(b_{k-1}-b_{k}).
$$
Now ${s_nb_n}$ is convergent, as a product of convergent sequences, while the series
$sum s_{n-1}(b_{n-1}-b_{n})$
is dominated by the series $sum M(b_{n}-b_{n-1})$, where $M=sup_{ninmathbb N}|s_n|$, and since ${b_{n}-b_{n-1}}$ is absolutely summable,
so is $sum s_{n-1}(b_{n-1}-b_{n})$.
$endgroup$
$begingroup$
yeah, it looks like a similar idea, stated brilliantly.
$endgroup$
– shota kobakhidze
Mar 15 at 19:11
add a comment |
$begingroup$
Assume that ${b_n}$ is increasing. If $s_n=sum_{k=1}^n a_k$, then
$$
sum_{k=1}^n a_kb_k=sum_{k=1}^n (s_k-s_{k-1})b_k=sum_{k=1}^n s_kb_k-sum_{k=2}^n s_{k-1}b_k=s_nb_n+
sum_{k=2}^{n}s_{k-1}(b_{k-1}-b_{k}).
$$
Now ${s_nb_n}$ is convergent, as a product of convergent sequences, while the series
$sum s_{n-1}(b_{n-1}-b_{n})$
is dominated by the series $sum M(b_{n}-b_{n-1})$, where $M=sup_{ninmathbb N}|s_n|$, and since ${b_{n}-b_{n-1}}$ is absolutely summable,
so is $sum s_{n-1}(b_{n-1}-b_{n})$.
$endgroup$
$begingroup$
yeah, it looks like a similar idea, stated brilliantly.
$endgroup$
– shota kobakhidze
Mar 15 at 19:11
add a comment |
$begingroup$
Assume that ${b_n}$ is increasing. If $s_n=sum_{k=1}^n a_k$, then
$$
sum_{k=1}^n a_kb_k=sum_{k=1}^n (s_k-s_{k-1})b_k=sum_{k=1}^n s_kb_k-sum_{k=2}^n s_{k-1}b_k=s_nb_n+
sum_{k=2}^{n}s_{k-1}(b_{k-1}-b_{k}).
$$
Now ${s_nb_n}$ is convergent, as a product of convergent sequences, while the series
$sum s_{n-1}(b_{n-1}-b_{n})$
is dominated by the series $sum M(b_{n}-b_{n-1})$, where $M=sup_{ninmathbb N}|s_n|$, and since ${b_{n}-b_{n-1}}$ is absolutely summable,
so is $sum s_{n-1}(b_{n-1}-b_{n})$.
$endgroup$
Assume that ${b_n}$ is increasing. If $s_n=sum_{k=1}^n a_k$, then
$$
sum_{k=1}^n a_kb_k=sum_{k=1}^n (s_k-s_{k-1})b_k=sum_{k=1}^n s_kb_k-sum_{k=2}^n s_{k-1}b_k=s_nb_n+
sum_{k=2}^{n}s_{k-1}(b_{k-1}-b_{k}).
$$
Now ${s_nb_n}$ is convergent, as a product of convergent sequences, while the series
$sum s_{n-1}(b_{n-1}-b_{n})$
is dominated by the series $sum M(b_{n}-b_{n-1})$, where $M=sup_{ninmathbb N}|s_n|$, and since ${b_{n}-b_{n-1}}$ is absolutely summable,
so is $sum s_{n-1}(b_{n-1}-b_{n})$.
answered Mar 15 at 19:03
Yiorgos S. SmyrlisYiorgos S. Smyrlis
63.6k1385165
63.6k1385165
$begingroup$
yeah, it looks like a similar idea, stated brilliantly.
$endgroup$
– shota kobakhidze
Mar 15 at 19:11
add a comment |
$begingroup$
yeah, it looks like a similar idea, stated brilliantly.
$endgroup$
– shota kobakhidze
Mar 15 at 19:11
$begingroup$
yeah, it looks like a similar idea, stated brilliantly.
$endgroup$
– shota kobakhidze
Mar 15 at 19:11
$begingroup$
yeah, it looks like a similar idea, stated brilliantly.
$endgroup$
– shota kobakhidze
Mar 15 at 19:11
add a comment |
$begingroup$
The general idea is correct. The expression “the upper bound of the sequence” is not appropriate (there are infinitely many upper bounds). I think that it would be better to say that $b_{up}$ is an upper bound of the sequence $bigl(lvert b_nrvertbigr)_{ninmathbb N}$. With this choice, $b_{up}$ is non-negative. Also, with this choice you have$$leftlvertsum_{n=p}^{q-1}s_n(b_n-b_{n-1})rightrvertleqslant2leftlvertsum_{n=p}^{q-1}s_nrightrvert b_{up}.$$Again, the expression “the difference of upper and lower bound of the sequence $b_n$” is not appropriate.
$endgroup$
$begingroup$
yeah, understood. Thanks for the response. By the way, saying that "we can make it as small as we want" is incorrect right? It would be more correct to say "bounded". Then the whole sum becomes "bounded" and not "as small as I wanted".
$endgroup$
– shota kobakhidze
Mar 15 at 18:38
$begingroup$
Oops! You are right. What you shoud use at that point is that $(s_n)_{ninmathbb N}$ is a boounded sequence and that the series $sum_{n=1}^inftylvert b_n-b_{n+1}rvert$ converges (this follows from the fact that $(b_n)_{ninmathbb N}$ is a monotonic and bounded sequence).
$endgroup$
– José Carlos Santos
Mar 15 at 18:47
$begingroup$
By the way, is it necessary for $b_{n}$ to be monotonic? cause I haven't used that fact, I just used the boundedness.
$endgroup$
– shota kobakhidze
Mar 15 at 23:16
$begingroup$
See my previous comment. Without that sequence being monotonic, how do you know that the series $sum_{n=1}^inftylvert b_n-b_{n+1}rvert$ converges?
$endgroup$
– José Carlos Santos
Mar 15 at 23:33
$begingroup$
yes, I see. It was my bad. Thanks :)
$endgroup$
– shota kobakhidze
Mar 16 at 0:00
add a comment |
$begingroup$
The general idea is correct. The expression “the upper bound of the sequence” is not appropriate (there are infinitely many upper bounds). I think that it would be better to say that $b_{up}$ is an upper bound of the sequence $bigl(lvert b_nrvertbigr)_{ninmathbb N}$. With this choice, $b_{up}$ is non-negative. Also, with this choice you have$$leftlvertsum_{n=p}^{q-1}s_n(b_n-b_{n-1})rightrvertleqslant2leftlvertsum_{n=p}^{q-1}s_nrightrvert b_{up}.$$Again, the expression “the difference of upper and lower bound of the sequence $b_n$” is not appropriate.
$endgroup$
$begingroup$
yeah, understood. Thanks for the response. By the way, saying that "we can make it as small as we want" is incorrect right? It would be more correct to say "bounded". Then the whole sum becomes "bounded" and not "as small as I wanted".
$endgroup$
– shota kobakhidze
Mar 15 at 18:38
$begingroup$
Oops! You are right. What you shoud use at that point is that $(s_n)_{ninmathbb N}$ is a boounded sequence and that the series $sum_{n=1}^inftylvert b_n-b_{n+1}rvert$ converges (this follows from the fact that $(b_n)_{ninmathbb N}$ is a monotonic and bounded sequence).
$endgroup$
– José Carlos Santos
Mar 15 at 18:47
$begingroup$
By the way, is it necessary for $b_{n}$ to be monotonic? cause I haven't used that fact, I just used the boundedness.
$endgroup$
– shota kobakhidze
Mar 15 at 23:16
$begingroup$
See my previous comment. Without that sequence being monotonic, how do you know that the series $sum_{n=1}^inftylvert b_n-b_{n+1}rvert$ converges?
$endgroup$
– José Carlos Santos
Mar 15 at 23:33
$begingroup$
yes, I see. It was my bad. Thanks :)
$endgroup$
– shota kobakhidze
Mar 16 at 0:00
add a comment |
$begingroup$
The general idea is correct. The expression “the upper bound of the sequence” is not appropriate (there are infinitely many upper bounds). I think that it would be better to say that $b_{up}$ is an upper bound of the sequence $bigl(lvert b_nrvertbigr)_{ninmathbb N}$. With this choice, $b_{up}$ is non-negative. Also, with this choice you have$$leftlvertsum_{n=p}^{q-1}s_n(b_n-b_{n-1})rightrvertleqslant2leftlvertsum_{n=p}^{q-1}s_nrightrvert b_{up}.$$Again, the expression “the difference of upper and lower bound of the sequence $b_n$” is not appropriate.
$endgroup$
The general idea is correct. The expression “the upper bound of the sequence” is not appropriate (there are infinitely many upper bounds). I think that it would be better to say that $b_{up}$ is an upper bound of the sequence $bigl(lvert b_nrvertbigr)_{ninmathbb N}$. With this choice, $b_{up}$ is non-negative. Also, with this choice you have$$leftlvertsum_{n=p}^{q-1}s_n(b_n-b_{n-1})rightrvertleqslant2leftlvertsum_{n=p}^{q-1}s_nrightrvert b_{up}.$$Again, the expression “the difference of upper and lower bound of the sequence $b_n$” is not appropriate.
edited Mar 15 at 19:16
J. W. Tanner
3,9221320
3,9221320
answered Mar 15 at 18:29
José Carlos SantosJosé Carlos Santos
170k23132239
170k23132239
$begingroup$
yeah, understood. Thanks for the response. By the way, saying that "we can make it as small as we want" is incorrect right? It would be more correct to say "bounded". Then the whole sum becomes "bounded" and not "as small as I wanted".
$endgroup$
– shota kobakhidze
Mar 15 at 18:38
$begingroup$
Oops! You are right. What you shoud use at that point is that $(s_n)_{ninmathbb N}$ is a boounded sequence and that the series $sum_{n=1}^inftylvert b_n-b_{n+1}rvert$ converges (this follows from the fact that $(b_n)_{ninmathbb N}$ is a monotonic and bounded sequence).
$endgroup$
– José Carlos Santos
Mar 15 at 18:47
$begingroup$
By the way, is it necessary for $b_{n}$ to be monotonic? cause I haven't used that fact, I just used the boundedness.
$endgroup$
– shota kobakhidze
Mar 15 at 23:16
$begingroup$
See my previous comment. Without that sequence being monotonic, how do you know that the series $sum_{n=1}^inftylvert b_n-b_{n+1}rvert$ converges?
$endgroup$
– José Carlos Santos
Mar 15 at 23:33
$begingroup$
yes, I see. It was my bad. Thanks :)
$endgroup$
– shota kobakhidze
Mar 16 at 0:00
add a comment |
$begingroup$
yeah, understood. Thanks for the response. By the way, saying that "we can make it as small as we want" is incorrect right? It would be more correct to say "bounded". Then the whole sum becomes "bounded" and not "as small as I wanted".
$endgroup$
– shota kobakhidze
Mar 15 at 18:38
$begingroup$
Oops! You are right. What you shoud use at that point is that $(s_n)_{ninmathbb N}$ is a boounded sequence and that the series $sum_{n=1}^inftylvert b_n-b_{n+1}rvert$ converges (this follows from the fact that $(b_n)_{ninmathbb N}$ is a monotonic and bounded sequence).
$endgroup$
– José Carlos Santos
Mar 15 at 18:47
$begingroup$
By the way, is it necessary for $b_{n}$ to be monotonic? cause I haven't used that fact, I just used the boundedness.
$endgroup$
– shota kobakhidze
Mar 15 at 23:16
$begingroup$
See my previous comment. Without that sequence being monotonic, how do you know that the series $sum_{n=1}^inftylvert b_n-b_{n+1}rvert$ converges?
$endgroup$
– José Carlos Santos
Mar 15 at 23:33
$begingroup$
yes, I see. It was my bad. Thanks :)
$endgroup$
– shota kobakhidze
Mar 16 at 0:00
$begingroup$
yeah, understood. Thanks for the response. By the way, saying that "we can make it as small as we want" is incorrect right? It would be more correct to say "bounded". Then the whole sum becomes "bounded" and not "as small as I wanted".
$endgroup$
– shota kobakhidze
Mar 15 at 18:38
$begingroup$
yeah, understood. Thanks for the response. By the way, saying that "we can make it as small as we want" is incorrect right? It would be more correct to say "bounded". Then the whole sum becomes "bounded" and not "as small as I wanted".
$endgroup$
– shota kobakhidze
Mar 15 at 18:38
$begingroup$
Oops! You are right. What you shoud use at that point is that $(s_n)_{ninmathbb N}$ is a boounded sequence and that the series $sum_{n=1}^inftylvert b_n-b_{n+1}rvert$ converges (this follows from the fact that $(b_n)_{ninmathbb N}$ is a monotonic and bounded sequence).
$endgroup$
– José Carlos Santos
Mar 15 at 18:47
$begingroup$
Oops! You are right. What you shoud use at that point is that $(s_n)_{ninmathbb N}$ is a boounded sequence and that the series $sum_{n=1}^inftylvert b_n-b_{n+1}rvert$ converges (this follows from the fact that $(b_n)_{ninmathbb N}$ is a monotonic and bounded sequence).
$endgroup$
– José Carlos Santos
Mar 15 at 18:47
$begingroup$
By the way, is it necessary for $b_{n}$ to be monotonic? cause I haven't used that fact, I just used the boundedness.
$endgroup$
– shota kobakhidze
Mar 15 at 23:16
$begingroup$
By the way, is it necessary for $b_{n}$ to be monotonic? cause I haven't used that fact, I just used the boundedness.
$endgroup$
– shota kobakhidze
Mar 15 at 23:16
$begingroup$
See my previous comment. Without that sequence being monotonic, how do you know that the series $sum_{n=1}^inftylvert b_n-b_{n+1}rvert$ converges?
$endgroup$
– José Carlos Santos
Mar 15 at 23:33
$begingroup$
See my previous comment. Without that sequence being monotonic, how do you know that the series $sum_{n=1}^inftylvert b_n-b_{n+1}rvert$ converges?
$endgroup$
– José Carlos Santos
Mar 15 at 23:33
$begingroup$
yes, I see. It was my bad. Thanks :)
$endgroup$
– shota kobakhidze
Mar 16 at 0:00
$begingroup$
yes, I see. It was my bad. Thanks :)
$endgroup$
– shota kobakhidze
Mar 16 at 0:00
add a comment |
Thanks for contributing an answer to Mathematics Stack Exchange!
- Please be sure to answer the question. Provide details and share your research!
But avoid …
- Asking for help, clarification, or responding to other answers.
- Making statements based on opinion; back them up with references or personal experience.
Use MathJax to format equations. MathJax reference.
To learn more, see our tips on writing great answers.
Sign up or log in
StackExchange.ready(function () {
StackExchange.helpers.onClickDraftSave('#login-link');
});
Sign up using Google
Sign up using Facebook
Sign up using Email and Password
Post as a guest
Required, but never shown
StackExchange.ready(
function () {
StackExchange.openid.initPostLogin('.new-post-login', 'https%3a%2f%2fmath.stackexchange.com%2fquestions%2f3149615%2frudin-exercise-3-8-solution%23new-answer', 'question_page');
}
);
Post as a guest
Required, but never shown
Sign up or log in
StackExchange.ready(function () {
StackExchange.helpers.onClickDraftSave('#login-link');
});
Sign up using Google
Sign up using Facebook
Sign up using Email and Password
Post as a guest
Required, but never shown
Sign up or log in
StackExchange.ready(function () {
StackExchange.helpers.onClickDraftSave('#login-link');
});
Sign up using Google
Sign up using Facebook
Sign up using Email and Password
Post as a guest
Required, but never shown
Sign up or log in
StackExchange.ready(function () {
StackExchange.helpers.onClickDraftSave('#login-link');
});
Sign up using Google
Sign up using Facebook
Sign up using Email and Password
Sign up using Google
Sign up using Facebook
Sign up using Email and Password
Post as a guest
Required, but never shown
Required, but never shown
Required, but never shown
Required, but never shown
Required, but never shown
Required, but never shown
Required, but never shown
Required, but never shown
Required, but never shown
$begingroup$
yeah, "making as small as we want" is a bad idea, but I can say that I can bound that sum by taking appropriate $p$ and $q$.
$endgroup$
– shota kobakhidze
Mar 15 at 18:28